User Avatar
JoshuaA1100
Joined
Jul 2025
Subscription
Free
PrepTests ·
PT137.S2.Q23
User Avatar
JoshuaA1100
Tuesday, Oct 28

This is a crazy question under timed conditions. I feel like parallel flaw questions sometimes punish you for treating them like parallel reasoning questions, so that makes me hesistant on choosing B just because of it's similar structure. A has a flaw that is quite similar to the stimulus. I feel like it is close enought to be a correct AC if B wasn't there.

PrepTests ·
PT137.S1.P2.Q10
User Avatar
JoshuaA1100
Sunday, Sep 28

Yeah this is wack. This passage does not indicate that seminomadic societies are simplistic. In paragraph 2 it even mentions the aspects of the !Kung that result in westerners' "received attitudes about 'simple' societies". Those aspects are not about seminomadism so we can't make the connection between seminomadism and simple societies as this question illicitly requires.

PrepTests ·
PT131.S2.Q18
User Avatar
JoshuaA1100
Tuesday, Oct 28

Kevin's explanation nails the error I made in this question. I conflated 'increasing food production' with 'agricultural advances' and thought that the sentence in question was starting off a causal chain that would end up supporting Malthus' conclusion.

PrepTests ·
PT158.S1.P4.Q23
User Avatar
JoshuaA1100
Monday, Oct 27

Language is too strong for this be the correct AC. There's a difference between something being 'inappropriately high' and another being able to better 'determine appropriate levels of damage'. If the stem said something like 'would most likely believe' that would have made this a more acceptable AC. But it just says they 'believe that'. We can't say this for sure given how strongly worded the language is.

PrepTests ·
PT148.S3.Q15
User Avatar
JoshuaA1100
Thursday, Sep 25

This was the only question I got wrong on this section because, like many here, I did not think intent was necessary for fraud. Funny for this to be a thing on LSAT because one would imagine that learning what is considered to be fraud would be something you learn in law school. It sounds like the subject of what I imagine a question on the bar exam would be.

The LSAT usually does a good job of providing all the info you need to know in the stimulus but I feel this question does demand a pre-existing conception of what constitutes fraud.

This question is interesting alongside PT149.S4.Q18. In that question, the necessary assumption requires you to know that belief is required for the intention to mislead. So taken from these 2 questions: Fraud→Intent→Belief. I think the link between Intent & Belief makes intuitive sense but Fraud→Intent is something that should be established in the stimulus.

If anyone knows any other questions that deal with intentions and what they imply/require I'd love to review them.

PrepTests ·
PT149.S3.Q22
User Avatar
JoshuaA1100
Wednesday, Sep 24

I chose D because it sucks. It does barely anything to resolve the paradox. We have to assume Charlesville residents are used to warmer temps. Then we have to assume that their keeping homes more than 10 degrees warmer on avg than those in Taychester. And 10 degrees is a large number in terms of home temperature in farenheight and an even larger number in celsius. However, B definitely is the right answer because there's no assumptions that can be made that would allow it to resolve the paradox.

PrepTests ·
PT156.S4.Q11
User Avatar
JoshuaA1100
Edited Sunday, Sep 21

Why is B necessary? If only some nations are required to give up industry then it still can be true that most nations would abide by the law. I do see now that the negation of B destroys the argument, but as the argument stands I cannot agree that it is necessary. The existance of 'some' and 'most' are not mutually exclusive

PrepTests ·
PT125.S2.Q23
User Avatar
JoshuaA1100
Saturday, Sep 20

Last Reviewed

This question is interesting because it features south a simple statement: A→B→C | TF A→C

But the wording of the ACs make it hard to decipher how to put these relationships into a conditional statement. What we should really be looking for to speed things up is a conclusion that gives us A→C. If we see other things we can eliminate

PrepTests ·
PT146.S3.Q25
User Avatar
JoshuaA1100
Saturday, Sep 20

I try not to diagram parallel flaw questions like I do parallel reasoning because the same flaw can occur with different terms. But for this one I had to diagram because the ACs seemed close to me.

As explained, E's major flaw is that it's conclusion is unrelated. But wouldn't it also be making a percentage vs amount flaw? It seems that's the reasoning they have for making the unrelated conclusion.

If I'm not mistaken about E also making a precentage vs absolute flaw, would it be correct to say that for parallel flaw questions we should eliminate an AC if it has a more glaring flaw than the one in the stimmy?

PrepTests ·
PT107.S3.Q19
User Avatar
JoshuaA1100
Friday, Sep 19

Nahhh. 'E' is not a necessary assumption. It could be the rocks melted as they burned up plummeting through the atmosphere. Then it would not be the impact causing them to melt but they would still crystalize at the same time as if they melted due to the impact. I chose this over 'A' because there's no point in making this argument if 'A' is not true. If the crater wasn't of a size to cause a mass extinction then that's your argument. Not this.

PrepTests ·
PT129.S4.P4.Q27
User Avatar
JoshuaA1100
Friday, Sep 19

This was a rough question for me. I was between 'B' and 'D'. I felt both were not proven. Author doesn't say that any breakthroughs weren't possible due to computers. But they do mention something that it 'dramatically illustrates'. For 'D' I didn't understand that proving theorems was an application. I was looking at the example of clouds as an application, but that hasn't happened yet so I thought there was no evidence of applications for fractals. So answering this right really depends on knowing what a mathmatical application & a theorem are, because neither are explained in the passage.

The actually useful takeaway from me is that it's better to select an AC that I'm confused about rather than one that I'm pretty sure has no evidence.

PrepTests ·
PT149.S2.P3.Q17
User Avatar
JoshuaA1100
Thursday, Sep 18

Sometimes I laugh at how bad some questions are. The benefit is from the existence of the norms and if other chefs are 'observing' the norm. Observing the norm confers the user with no benefit. Any chef can violate the norm regardless if one observes it or not. I hope I can at least learn from this question not choose a question that says 'most' when no quantifiers for a population are mentioned. I almost chose E but it seemed so obviously wrong when no benefits/sanctions are mentioned in passage B.

PrepTests ·
PT146.S2.Q1
User Avatar
JoshuaA1100
Wednesday, Sep 17

D seemed so obviously right I was almost too afraid to select it

PrepTests ·
PT153.S1.P2.Q11
User Avatar
JoshuaA1100
Wednesday, Sep 17

I got fooled into eliminating 'D' because it refers to a decrease in food type. To me, the issue wasn't a decrease in food, it was a decrease in fish. Sure, fish can be food but I thought it was more out of ecological concern. However, back in paragraph 1 the author makes it clear that they're worried about fishery stocks being depleted and there not being enough fish for human consumption

PrepTests ·
PT130.S4.Q20
User Avatar
JoshuaA1100
Wednesday, Sep 17

'B' does not require that you assume the other dietary factors are more important to increasing BP. There could any number of dietary factors that impact BP. Even if salt is the most significant, if the other factors have a low intake and salt has a high intake, one's BP could still be very low. And still, for those with high BP, decreasing salt intake would still result in an overall decrease in BP. This addresses both aspects of this paradox so why is wrong over 'E' which has 3 major problems.

Assumption Festival - Assumptions required by 'E'

1) Very low BP & abnormal BP are the same. We don't know this, it's not implied, so it's an assumption. Something being 'abnormal' means that it is not the norm. But we don't know what the norm is. Very low BP might not be abnormal and therefore. There's also the assumption

2) Heightened salt cravings = very high salt intake. Just because one craves something doesn't mean one consumes it.

3) Assumes 'Some' can generalize to an entire population. Probably the worst flaw of this AC. The stimmy tells us that some people with very high salt intake have very low BP. We know from so many other questions on the LSAT that 'some' of a population cannot be used to make a generalization about an entire pop. So why can 'E' do it?

PrepTests ·
PT124.S4.P4.Q24
User Avatar
JoshuaA1100
Monday, Sep 15

This is a terrible question. The parathion example is just 1 example of a case in which insecticides shouldn't be used. This example features only 1 type of prey, 1 type of predator, and 1 type of insecsticide. Furthermore, the author never advocates for the usage of insecticides. There could be plenty of other situations in which insecticides may be useful alongside, or in lieu, of predators. I really think 'Only if' is just too strong of language to justify an inference about the author when the only opinion they have is that insecticides shouldn't be used in this 1 example.

PrepTests ·
PT116.S3.Q25
User Avatar
JoshuaA1100
Edited Monday, Sep 15

Criminal usage of 'often'. But there's 2 lesson I got from this.

1) 'Often' = 'some'

2) Pay closer attention to the context of words like 'some' & 'most' as they might be required to get left out your diagram

PrepTests ·
PT155.S2.Q22
User Avatar
JoshuaA1100
Saturday, Sep 13

Great explanation from J.Y. here. There's a lot to learn from analyzing this question and its ACs. Correct me if I'm wrong but if diagrammed the stimulus reads:

A(most)B→C | (most) C.

B has the same structure but with 'some' instead and J.Y. explains how you can make that conclusion because 'some' has a lesser burden than 'most' has. And it would be valid if the stimulus argued:

A(most)B→C | A(most) C.

This would be valid because A introduced the domain to limit the population of C we must consider. It's really hard though to parse through these ACs in timed conditions but practicing on questions like this is helpful for recognizing the more rare conditional structures.

PrepTests ·
PT124.S3.Q22
User Avatar
JoshuaA1100
Friday, Sep 12

Ahhhh man, I think I eliminated B in my head before I even finished reading. That last word carries a LOT of weight

PrepTests ·
PT136.S1.P1.Q1
User Avatar
JoshuaA1100
Wednesday, Sep 10

Main point questions in LR are easy. In RC they are terrifying.

I just took PT 148 Section 1 and I'm very confused with question 21. It reads:

No occupation should be subject to a licensing requirement unless incompetence in the performance of tasks normally carried out within that occupation poses a plausible threat to human health or safety.

'No' is a negated necessary indicator & 'unless' is a negated sufficient indicator. The curriculum says when encountering these it doesn't matter if you put the term as the necessary or sufficient condition because of the contrapositive. But, when you have both indicators that surely can't be the case as this question proves. If the curriculm covers how to handle conditional statement with both kinds of indicators I missed it.

So my question is, how do you handle these? Do you just need to use intuition to figure out which term goes in the sufficient/necessary spots? Do you prioritize one indicator over the other? Or do the indicators cancel each other out and you proceed as if they never existed?

The diagram for this questions is

If subject to licensing agreement (LR) → Incompetence poses threat (IPT)

(Contra) /IPT → /LR

PrepTests ·
PT106.S2.Q19
User Avatar
JoshuaA1100
Edited Wednesday, Oct 29

WTFFF. Someone please explain to me why D must be true. Why can't it be true that most mail that is correctly addressed is damaged in transit? We have no info about how much mail is damaged in transit or how much of it is incorrectly addressed. To me, the same reason why A is wrong is why D is wrong

PrepTests ·
PT157.S1.P3.Q18
User Avatar
JoshuaA1100
Edited Thursday, Oct 02

This passage is nuts! It has three 180 curvebreaker questions. I only got 1 of them right. I had no idea how to approach this question. I don't think I've ever seen a question like this and had no idea what they were looking for by an 'appropriate response'? I chose D because it would have been what best represented passage A's interests which would seem appropriate to them.

PrepTests ·
PT157.S1.P2.Q9
User Avatar
JoshuaA1100
Thursday, Oct 02

I am not a fan of this one. 'D' starts off sounding really good, having no proof of something is exactly what faith is. But the 'prescriptivists' efforts have no effect on the evolution of language' part sounded far too strong. Funny that I eliminated the correct answer for being too strong and chose a wrong answer that was also too strong.

In 'D' the author concedes 2 sentences before this phrase that 'many attempts to regulate language have failed' so that sounds like some evidence to me. And what about the perscriptivists efforts? This part of the passage is referring to the descriptivists denying that some perscriptivists efforts invalidate the scientific objection. This is a very narrow and specific effort from the perscriptivists. To say that from this, none of their efforts have had any effect on the evolution of language is flawed reasoning.

However, the question is asking us about the author's intent on using this phrase. While it may be true that this claim is too strong, it doesn't mean the author didn't use the phrase in that manner.

Lesson: Focus on author's intent. This isn't an MBT or flaw question. RC is something else.

PrepTests ·
PT103.S2.Q25
User Avatar
JoshuaA1100
Sunday, Nov 02

A does not make sense to me. This seems like a sufficient assumption but it seems too strong to be a necessary assumption. What if the public's opinion will lead to physical harm? Should it then be enacted? Changing this art won't reasonably lead to any physical harm so Hector makes no mention of this consideration. But since he does not take this into consideration we can't say this statement is necessary. The 'no matter what' in this AC opens it up to any possible situation. What am I missing here?

PrepTests ·
PT103.S2.Q22
User Avatar
JoshuaA1100
Edited Saturday, Nov 01

E makes me sad. I chose it because I thought, when it said 'courses' the second time, that it was referring to courses in general, from any subject. My reasoning was that perhaps there are other subjects in which stats and dates wouldn't be so boring. With this interpretation, the AC is not necessary.

I just can't seem to get these 5 star NA questions right. There's often something that I'm uncertain about in an AC whichs leds me to an incorrect AC

Confirm action

Are you sure?